Aufgaben:Aufgabe 3.3Z: Rechteck- und Diracimpuls: Unterschied zwischen den Versionen

Aus LNTwww
Wechseln zu:Navigation, Suche
Zeile 4: Zeile 4:
  
 
[[Datei:P_ID507__Sig_Z_3_3.png|right|frame|Verschiedene Rechteckimpulse]]
 
[[Datei:P_ID507__Sig_Z_3_3.png|right|frame|Verschiedene Rechteckimpulse]]
Wir betrachten hier eine Vielzahl von symmetrischen Rechteckfunktionen $x_k(t)$. Die einzelnen Rechtecke unterscheiden sich durch unterschiedliche Amplituden (Höhen)
+
Wir betrachten hier eine Vielzahl von symmetrischen Rechteckfunktionen  $x_k(t)$. Die einzelnen Rechtecke unterscheiden sich durch unterschiedliche Amplituden (Höhen)
 
:$$A_k  = k \cdot A$$
 
:$$A_k  = k \cdot A$$
 
und unterschiedliche Impulsdauern (Breiten)
 
und unterschiedliche Impulsdauern (Breiten)
 
:$$T_k = T/k.$$
 
:$$T_k = T/k.$$
Hierbei sei $k$ ein beliebiger positiver Wert.  
+
Hierbei sei  $k$  ein beliebiger positiver Wert.  
 +
 
 +
*Der im Bild rot dargestellte Rechteckimpuls  $x_1(t)$  hat die Amplitude  $A_1 = {A} = 2 \,\text{V}$  und die Dauer  $T_1 = {T} = 500 \,µ\text{s}$.
 +
*Der blau gezeichnete Impuls  $x_2(t)$  ist halb so breit   ⇒   $T_2 =250 \,µ\text{s}$, aber doppelt so hoch   ⇒   $A_2 = 4 \text{ V}$.
 +
 
 +
 
  
*Der im Bild rot dargestellte Rechteckimpuls $x_1(t)$ hat die Amplitude $A_1 = {A} = 2 \,\text{V}$ und die Dauer $T_1 = {T} = 500 \,µ\text{s}$.
 
*Der blau gezeichnete Impuls $x_2(t)$ ist halb so breit   ⇒   $T_2 =250 \,µ\text{s}$, aber doppelt so hoch   ⇒   $A_2 = 4 \text{ V}$.
 
  
  
Zeile 18: Zeile 21:
  
 
''Hinweise:''  
 
''Hinweise:''  
*Die Aufgabe gehört zum  Kapitel [[Signaldarstellung/Einige_Sonderfälle_impulsartiger_Signale|Einige Sonderfälle impulsartiger Signale]].
+
*Die Aufgabe gehört zum  Kapitel  [[Signaldarstellung/Einige_Sonderfälle_impulsartiger_Signale|Einige Sonderfälle impulsartiger Signale]].
 
   
 
   
*Sie können Ihre Ergebnisse anhand der beiden  interaktiven Applets [[Applets:Impulse_und_Spektren|Impulse und Spektren]] sowie  [[Applets:Frequenzgang_und_Impulsantwort|Frequenzgang und Impulsantwort]] überprüfen.
+
*Sie können Ihre Ergebnisse anhand der beiden  interaktiven Applets  [[Applets:Impulse_und_Spektren|Impulse und Spektren]]  sowie  [[Applets:Frequenzgang_und_Impulsantwort|Frequenzgang und Impulsantwort]]  überprüfen.
  
  
Zeile 28: Zeile 31:
  
 
<quiz display=simple>
 
<quiz display=simple>
{Welche der folgenden Aussagen treffen bezüglich des Spektrums $X_1(f)$ zu?
+
{Welche der folgenden Aussagen treffen bezüglich des Spektrums&nbsp; $X_1(f)$&nbsp; zu?
 
|type="[]"}
 
|type="[]"}
+ Der Spektralwert $X_1(f = 0)$ ist gleich $10^{–3} \,\text{V/Hz}$.
+
+ Der Spektralwert&nbsp; $X_1(f = 0)$&nbsp; ist gleich&nbsp; $10^{–3} \,\text{V/Hz}$.
+ $X_1(f)$ besitzt Nullstellen im Abstand von $2 \,\text{kHz}$.
+
+ $X_1(f)$&nbsp; besitzt Nullstellen im Abstand von&nbsp; $2 \,\text{kHz}$.
- $X_1(f)$ besitzt Nullstellen im Abstand von $4 \,\text{kHz}$.
+
- $X_1(f)$&nbsp; besitzt Nullstellen im Abstand von&nbsp; $4 \,\text{kHz}$.
  
  
{Welche der folgenden Aussagen treffen bezüglich des Spektrums $X_2(f)$ zu?
+
{Welche der folgenden Aussagen treffen bezüglich des Spektrums&nbsp; $X_2(f)$&nbsp; zu?
 
|type="[]"}
 
|type="[]"}
+ Der Spektralwert $X_2(f = 0)$ ist gleich $10^{–3} \,\text{V/Hz}$.
+
+ Der Spektralwert&nbsp; $X_2(f = 0)$&nbsp; ist gleich&nbsp; $10^{–3} \,\text{V/Hz}$.
- $X_2(f)$ besitzt Nullstellen im Abstand von $2\, \text{kHz}$.
+
- $X_2(f)$&nbsp; besitzt Nullstellen im Abstand von&nbsp; $2\, \text{kHz}$.
+ $X_2(f)$ besitzt Nullstellen im Abstand von $4 \,\text{kHz}$.
+
+ $X_2(f)$&nbsp; besitzt Nullstellen im Abstand von&nbsp; $4 \,\text{kHz}$.
  
  
{Es gelte $k = 10$. Berechnen Sie die Frequenz $f_{10}$ der ersten Nullstelle und den Spektralwert bei $f = 2 \,\text{kHz}$.
+
{Es gelte&nbsp; $k = 10$. Berechnen Sie die Frequenz&nbsp; $f_{10}$&nbsp; der ersten Nullstelle und den Spektralwert bei&nbsp; $f = 2 \,\text{kHz}$.
 
|type="{}"}
 
|type="{}"}
 
$f_{10} \ = \ ${ 20 3% } &nbsp;$\text{kHz}$
 
$f_{10} \ = \ ${ 20 3% } &nbsp;$\text{kHz}$
Zeile 48: Zeile 51:
  
  
{Wie groß wird der Spektralwert bei $f = 2 \,\text{kHz}$ im Grenzfall $k \rightarrow \infty$? Interpretieren Sie das Ergebnis.
+
{Wie groß wird der Spektralwert&nbsp; bei $f = 2 \,\text{kHz}$&nbsp; im Grenzfall&nbsp; $k \rightarrow \infty$? Interpretieren Sie das Ergebnis.
 
|type="{}"}
 
|type="{}"}
 
$X_{\infty}(f = 2 \,\text{kHz})\ = \ $ { 1 3% } &nbsp;$\text{mV/Hz}$
 
$X_{\infty}(f = 2 \,\text{kHz})\ = \ $ { 1 3% } &nbsp;$\text{mV/Hz}$

Version vom 26. September 2019, 11:52 Uhr

Verschiedene Rechteckimpulse

Wir betrachten hier eine Vielzahl von symmetrischen Rechteckfunktionen  $x_k(t)$. Die einzelnen Rechtecke unterscheiden sich durch unterschiedliche Amplituden (Höhen)

$$A_k = k \cdot A$$

und unterschiedliche Impulsdauern (Breiten)

$$T_k = T/k.$$

Hierbei sei  $k$  ein beliebiger positiver Wert.

  • Der im Bild rot dargestellte Rechteckimpuls  $x_1(t)$  hat die Amplitude  $A_1 = {A} = 2 \,\text{V}$  und die Dauer  $T_1 = {T} = 500 \,µ\text{s}$.
  • Der blau gezeichnete Impuls  $x_2(t)$  ist halb so breit   ⇒   $T_2 =250 \,µ\text{s}$, aber doppelt so hoch   ⇒   $A_2 = 4 \text{ V}$.





Hinweise:



Fragebogen

1

Welche der folgenden Aussagen treffen bezüglich des Spektrums  $X_1(f)$  zu?

Der Spektralwert  $X_1(f = 0)$  ist gleich  $10^{–3} \,\text{V/Hz}$.
$X_1(f)$  besitzt Nullstellen im Abstand von  $2 \,\text{kHz}$.
$X_1(f)$  besitzt Nullstellen im Abstand von  $4 \,\text{kHz}$.

2

Welche der folgenden Aussagen treffen bezüglich des Spektrums  $X_2(f)$  zu?

Der Spektralwert  $X_2(f = 0)$  ist gleich  $10^{–3} \,\text{V/Hz}$.
$X_2(f)$  besitzt Nullstellen im Abstand von  $2\, \text{kHz}$.
$X_2(f)$  besitzt Nullstellen im Abstand von  $4 \,\text{kHz}$.

3

Es gelte  $k = 10$. Berechnen Sie die Frequenz  $f_{10}$  der ersten Nullstelle und den Spektralwert bei  $f = 2 \,\text{kHz}$.

$f_{10} \ = \ $

 $\text{kHz}$
$X_{10}(f = 2 \text{kHz})\ = \ $

 $\text{mV/Hz}$

4

Wie groß wird der Spektralwert  bei $f = 2 \,\text{kHz}$  im Grenzfall  $k \rightarrow \infty$? Interpretieren Sie das Ergebnis.

$X_{\infty}(f = 2 \,\text{kHz})\ = \ $

 $\text{mV/Hz}$


Musterlösung

(1)  Richtig sind die Lösungsvorschläge 1 und 2:

  • Der Spektralwert bei der Frequenz $f = 0$ ist nach dem ersten Fourierintegral stets gleich der Fläche unter der Zeitfunktion:
$$X( f ) = \int_{ - \infty }^{ + \infty } {x( t )} \cdot {\rm{e}}^{ - {\rm{j2\pi }}ft} \hspace{0.1cm} {\rm d}t \hspace{0.5cm} \Rightarrow \hspace{0.5cm} \;X( {f = 0} ) = \int_{ - \infty }^{ + \infty } {x( t )}\hspace{0.1cm} {\rm d}t.$$
  • Im vorliegenden Fall ist die Impulsfläche stets $A \cdot T = 10^{–3} \,\text{Vs} = 1\, \text{mV/Hz}$.
  • Wegen $T_1 = 500 \,µ\text{s}$ weist das Spektrum $X_1(f)$ Nulldurchgänge im Abstand $f_1 = 1/T_1 = 2 \,\text{kHz}$ auf.


(2)  Richtig sind die Lösungsvorschläge 1 und 3:

  • Aufgrund gleicher Impulsflächen wird der Spektralwert bei der Frequenz $f = 0$ nicht verändert.
  • Die äquidistanten Nulldurchgänge treten nun im Abstand $f_2 = 1/T_2 = 4 \,\text{kHz}$ auf.


(3)  Nullstellen gibt es bei Vielfachen von $f_{10} = 1/T_{10} = 20 \,\text{kHz}$, und die Spektralfunktion lautet:

$$X_{10} ( f ) = X_0 \cdot {\mathop{\rm si}\nolimits} ( {{\rm{\pi }}f/f_{10} } ).$$

Bei der Frequenz $f = 2 \,\text{kHz}$ ist das Argument der $\rm si$-Funktion gleich $\pi/10$ (oder $18^{\circ}$):

$$X_{10} ( {f = 2\;{\rm{kHz}}}) = 10^{ - 3} \;{\rm{V/Hz}} \cdot \frac{{\sin ( {18^\circ } )}}{{{\rm{\pi /10}}}} \hspace{0.15 cm}\underline{= 0.984 \;{\rm{mV/Hz}}}{\rm{.}}$$

(4)  Im Grenzfall $k \rightarrow \infty$ geht der dann unendlich hohe und unendlich schmale Rechteckimpuls in den Diracimpuls über. Dessen Spektrum ist für alle Frequenzen konstant. Damit gilt auch bei der Frequenz $f = 2 \,\text{kHz}$ der Spektralwert $X_{\infty}(f = 2 \,\text{kHz})\underline{=1 \text{ mV/Hz}}$.